nierównosc

Własności funkcji trygonometrycznych i cyklometrycznych. Tożsamości. RÓWNANIA I NIERÓWNOŚCI.
robin5hood
Użytkownik
Użytkownik
Posty: 1676
Rejestracja: 2 kwie 2007, o 14:43
Płeć: Mężczyzna
Lokalizacja: warszawa
Podziękował: 178 razy
Pomógł: 17 razy

nierównosc

Post autor: robin5hood »

rozwiązac nierównosc
\(\displaystyle{ sin(lnx)+cos(lnx)>-1,5}\)
Awatar użytkownika
Lider_M
Użytkownik
Użytkownik
Posty: 867
Rejestracja: 6 maja 2005, o 12:50
Płeć: Mężczyzna
Lokalizacja: MiNI PW
Pomógł: 258 razy

nierównosc

Post autor: Lider_M »

Podstaw najpierw \(\displaystyle{ \ln x=t}\), zamień np. cosinus na sinus ze wzorów redukcyjnych i zastosuj wzór na sumę sinusów.
robin5hood
Użytkownik
Użytkownik
Posty: 1676
Rejestracja: 2 kwie 2007, o 14:43
Płeć: Mężczyzna
Lokalizacja: warszawa
Podziękował: 178 razy
Pomógł: 17 razy

nierównosc

Post autor: robin5hood »

mozesz to rozwiazac do konca bo ja probuje i nic
Awatar użytkownika
Lider_M
Użytkownik
Użytkownik
Posty: 867
Rejestracja: 6 maja 2005, o 12:50
Płeć: Mężczyzna
Lokalizacja: MiNI PW
Pomógł: 258 razy

nierównosc

Post autor: Lider_M »

założenie \(\displaystyle{ x>0}\), dalej:
\(\displaystyle{ \sin t+\cos t>-\frac{3}{2} \\

\cos\left(t-\frac{\pi}{2}\right)+\cos t>-\frac{3}{2} \\

2\cos\frac{\pi}{4}\cos\left(t-\frac{\pi}{4}\right)>-\frac{3}{2} \\

\cos\left(t-\frac{\pi}{4}\right)>-\frac{3}{2\sqrt{2}}=-\frac{3\sqrt{2}}{4}}\)


Teraz posługujemy sie troche wykresem, i dalej mamy:

\(\displaystyle{ t-\frac{\pi}{4}\in\left(-\arccos\frac{-3\sqrt{2}}{4}+2k\pi;\arccos\frac{-3\sqrt{2}}{4}+2k\pi\right)}\) dla \(\displaystyle{ k\in\mathbb{Z}}\), no i dalej już tylko \(\displaystyle{ x}\)wyznaczyć.
Awatar użytkownika
max
Użytkownik
Użytkownik
Posty: 3306
Rejestracja: 10 gru 2005, o 17:48
Płeć: Mężczyzna
Lokalizacja: Lebendigentanz
Podziękował: 37 razy
Pomógł: 778 razy

nierównosc

Post autor: max »

Warto byłoby po drodze zauważyć, że:
\(\displaystyle{ -\frac{3\sqrt{2}}{4} < -1}\)
robin5hood
Użytkownik
Użytkownik
Posty: 1676
Rejestracja: 2 kwie 2007, o 14:43
Płeć: Mężczyzna
Lokalizacja: warszawa
Podziękował: 178 razy
Pomógł: 17 razy

nierównosc

Post autor: robin5hood »

i jaki z tego wniosek?
Awatar użytkownika
max
Użytkownik
Użytkownik
Posty: 3306
Rejestracja: 10 gru 2005, o 17:48
Płeć: Mężczyzna
Lokalizacja: Lebendigentanz
Podziękował: 37 razy
Pomógł: 778 razy

nierównosc

Post autor: max »

Badana nierówność zajdzie dla każdego \(\displaystyle{ t\in \mathbb{R}}\), bo dla każdego \(\displaystyle{ t\in\mathbb{R}}\) jest przecież:
\(\displaystyle{ \cos (t - \tfrac{\pi}{4}) \geqslant -1 > -\tfrac{3\sqrt{2}}{4}}\)
Stąd \(\displaystyle{ x\in (0, +\infty)}\).
ODPOWIEDZ